Are you seeking one-on-one college counseling and/or essay support? Limited spots are now available. Click here to learn more.

How to Write the AP Lang Argument Essay (With Example)

December 14, 2023

We’d like to let you in on a little secret: no one, including us, enjoys writing timed essays. But a little practice goes a long way. If you want to head into your AP English Exam with a cool head, you’ll want to know what you’re getting into ahead of time. We can’t promise the AP Lang Argument Essay will ever feel like an island vacation, but we do have tons of hand tips and tricks (plus a sample essay!) below to help you do your best. This article will cover: 1) What is the AP Lang Argumentative Essay? 2) AP Lang Argument Rubric 3) AP Lang Argument Sample Prompt 4) AP Lang Argument Essay Example 5) AP Lang Argument Essay Example: Answer Breakdown.

What is the AP Lang Argument Essay?

The AP Lang Argument Essay is one of three essays included in the written portion of the AP English Exam. The full AP English Exam is 3 hours and 15 minutes long, with the first 60 minutes dedicated to multiple-choice questions. Once you complete the multiple-choice section, you move on to three equally weighted essays that ask you to synthesize, analyze, and interpret texts and develop well-reasoned arguments. The three essays include:

Synthesis essay: You’ll review various pieces of evidence and then write an essay that synthesizes (aka combines and interprets) the evidence and presents a clear argument. Read our write-up on How to Write the AP Lang Synthesis Essay here.

Argumentative essay: You’ll take a stance on a specific topic and argue your case.

Rhetorical essay: You’ll read a provided passage, then analyze the author’s rhetorical choices and develop an argument that explains why the author made those rhetorical choices. Read our write-up on How to Write the AP Lang Rhetorical Essay here.

AP Lang Argument Essay Rubric

The AP Lang Argument Essay is graded on 3 rubric categories : Thesis, Evidence and Commentary, and Sophistication . How can you make sure you cover all three bases in your essay? We’ll break down each rubric category with dos and don’ts below:

  • Thesis (0-1 point)

When it comes to grading your thesis, AP Exam graders are checking off a box: you either have a clear thesis or you don’t. So, what crucial components of a thesis will get you your check mark?

  • Make sure your thesis argues something . To satisfy your graders, your thesis needs to take a clear stance on the issue at hand.
  • Include your thesis statement in your intro paragraph. The AP Lang Argumentative essay is just that: an essay that makes an argument, so make sure you present your argument right away at the end of your first paragraph.
  • A good test to see if you have a thesis that makes an argument for your AP Lang Argumentative Essay: In your head, add the phrase “I agree/disagree that…” to the beginning of your thesis. If what follows doesn’t logically flow after that phrase (aka if what follows isn’t an agreement or disagreement), it’s likely you’re not making an argument.
  • In your thesis, outline the evidence you’ll cover in your body paragraphs.

AP Lang Argument Essay Rubric (Continued)

  • Avoid a thesis that merely restates the prompt.
  • Avoid a thesis that summarizes the text but does not make an argument.
  • Avoid a thesis that weighs the pros and cons of an issue. Your job in your thesis is to pick a side and stick with it.
  • Evidence and Commentary (0-4 points)

This rubric category is graded on a scale of 0-4 where 4 is the highest grade. Unlike the rhetorical and synthesis essays, the evidence you need to write your AP Lang Argument Essay is not provided to you. Rather, you’ll need to generate your own evidence and comment upon it.

What counts as evidence?

Typically, the AP Lang Argument Essay prompt asks you to reflect on a broad cultural, moral, or social issue that is open to debate. For evidence, you won’t be asked to memorize and cite statistics or facts. Rather, you’ll want to bring in real-world examples of:

  • Historical events
  • Current-day events from the news
  • Personal anecdotes

For this essay, your graders know that you’re not able to do research to find the perfect evidence. What’s most important is that you find evidence that logically supports your argument.

What is commentary?

In this essay, it’s important to do more than just provide examples relevant evidence. After each piece of evidence you include, you’ll need to explain why it’s significant and how it connects to your main argument. The analysis you include after your evidence is commentary .

  • Take a minute to brainstorm evidence that logically supports your argument. If you have to go out of your way to find the connection, it’s better to think of different evidence.
  • Include multiple pieces of evidence. There is no magic number, but do make sure you incorporate more than a couple pieces of evidence that support your argument.
  • Make sure you include more than one example of evidence, too. Let’s say you’re working on an essay that argues that people are always stronger together than apart. You’ve already included an example from history: during the civil rights era, protestors staged group sit-ins as a powerful form of peaceful protest. That’s just one example, and it’s hard to make a credible argument with just one piece of evidence. To fix that issue, think of additional examples from history, current events, or personal experience that are not related to the civil rights era.
  • After you include each piece of evidence, explain why it’s significant and how it connects to your main argument.
  • Don’t summarize or speak generally about the topic. Everything you write must be backed up with specific and relevant evidence and examples.
  • Don’t let quotes speak for themselves. After every piece of evidence you include, make sure to explain and connect the evidence to your overarching argument.

AP Lang Argument Essay (Continued)

  • Sophistication (0-1 point)

According to the College Board , one point can be awarded to AP Lang Argument essays that achieve a high level of sophistication. You can accomplish that in four ways:

  • Crafting a nuanced argument by consistently identifying and exploring complexities or tensions.
  • Articulating the implications or limitations of an argument by situating it within a broader context.
  • Making effective rhetorical choices that consistently strengthen the force and impact of the student’s argument.
  • Employing a style that is consistently vivid and persuasive.

In sum, this means you can earn an additional point for going above and beyond in depth, complexity of thought, or by writing an especially persuasive, clear, and well-structured essay. In order to earn this point, you’ll first need to do a good job with the fundamentals: your thesis, evidence, and commentary. Then, to earn your sophistication point, follow these tips:

  • Outline your essay before you begin to ensure it flows in a clear and cohesive way.
  • Include well-rounded evidence. Don’t rely entirely on personal anecdotes, for example. Incorporate examples from current events or history, as well.
  • Thoroughly explain how each piece of evidence connects to your thesis in order to fully develop your argument.
  • Explore broader implications. If what you’re arguing is true, what does that mean to us today? Who is impacted by this issue? What real-world issues are relevant to this core issue?
  • Briefly explore the other side of the issue. Are the instances where your argument might not be true? Acknowledge the other side, then return to proving your original argument.
  • Steer clear of generalizations (avoid words like “always” and “everyone”).
  • Don’t choose an argument you can’t back up with relevant examples.
  • Avoid complex sentences and fancy vocabulary words unless you use them often. Long, clunky sentences with imprecisely used words are hard to follow.

AP Lang Argument Sample Prompt

The sample prompt below is published online by the College Board and is a real example from the 2021 AP English Exam. The prompt provides background context, essay instructions, and the text you need to analyze.

Suggested time—40 minutes.

Many people spend long hours trying to achieve perfection in their personal or professional lives. Similarly, people often demand perfection from others, creating expectations that may be challenging to live up to. In contrast, some people think perfection is not attainable or desirable.

Write an essay that argues your position on the value of striving for perfection.

In your response you should do the following:

  • Respond to the prompt with a thesis that presents a defensible position.
  • Provide evidence to support your line of reasoning.
  • Explain how the evidence supports your line of reasoning.
  • Use appropriate grammar and punctuation in communicating your argument.

AP Lang Argument Essay Example

As the old phrase says, “Practice makes perfect.” But is perfection something that is actually attainable? Sometimes, pushing for perfection helps us achieve great things, but most often, perfectionism puts too much pressure on us and prevents us from knowing when we have done the best we can. Striving for perfection can only lead us to shortchange ourselves. Instead, we should value learning, growth, and creativity and not worry whether we are first or fifth best.

Students often feel the need to be perfect in their classes, and this can cause students to struggle or stop making an effort in class. In elementary and middle school, for example, I was very nervous about public speaking. When I had to give a speech, my voice would shake, and I would turn very red. My teachers always told me “relax!” and I got Bs on Cs on my speeches. As a result, I put more pressure on myself to do well, spending extra time making my speeches perfect and rehearsing late at night at home. But this pressure only made me more nervous, and I started getting stomach aches before speaking in public.

Once I got to high school, however, I started doing YouTube make-up tutorials with a friend. We made videos just for fun, and laughed when we made mistakes or said something silly. Only then, when I wasn’t striving to be perfect, did I get more comfortable with public speaking.

AP Lang Argumentative Essay Example (Continued)

In the world of art and business and science, perfectionism can also limit what we are able to achieve. Artists, for example, have to take risks and leave room for creativity. If artists strive for perfection, then they won’t be willing to fail at new experiments and their work will be less innovative and interesting. In business and science, many products, like penicillin for example, were discovered by accident. If the scientist who discovered penicillin mold growing on his petri dishes had gotten angry at his mistake and thrown the dishes away, he would never have discovered a medicine that is vital to us today.

Some fields do need to value perfection. We wouldn’t like it, for example, if our surgeon wasn’t striving for perfection during our operation. However, for most of us, perfectionism can limit our potential for learning and growth. Instead of trying to be perfect, we should strive to learn, innovate, and do our personal best.

AP Lang Argument Essay Example: Answer Breakdown

The sample AP Lang Argumentative Essay above has some strengths and some weaknesses. Overall, we would give this essay a 3 or a 4. Let’s break down what’s working and what could be improved:

  • The essay offers a thesis that makes a clear argument that is relevant to the prompt: “Striving for perfection can only lead us to shortchange ourselves. Instead, we should value learning, growth, and creativity and not worry whether we are first or fifth best.”
  • The first body paragraph provides evidence that supports the essay’s thesis. This student’s personal anecdote offers an example of a time when perfectionism led them to shortchange themselves.
  • The second body paragraph provides additional evidence that supports the essay’s thesis. The example describing the discovery of penicillin offers another example of a situation in which perfectionism might have limited scientific progress.
  • The writer offers commentary explaining how her examples of public speaking and penicillin illustrate that we should “value learning, growth, and creativity” over perfectionism.
  • The essay follows one line of reasoning and does not stray into tangents.
  • The essay is organized well with intro, body, and concluding paragraphs. Overall, it is easy to read and is free of grammar errors.

What could be improved:

  • Although the second body paragraph provides one good specific example about the discovery of penicillin, the other examples it offers about art and business are only discussed generally and aren’t backed up with evidence. This paragraph would be stronger if it provided more examples. Or, if this writer couldn’t think of examples, they could have left out mentions of art and business altogether and included alternate evidence instead.
  • This writer would more thoroughly support their argument if they were able to offer one more example of evidence. They could provide another personal anecdote, an example from history, or an example from current events.
  • The writer briefly mentions the other side of the argument in their concluding paragraph: “Some fields do need to value perfection. We wouldn’t like it, for example, if our surgeon wasn’t striving for perfection during our operation.” Since it’s so brief a mention of the other side, it undermines the writer’s overall argument. This writer should either dedicate more time to reflecting on why even surgeons should “value learning, growth, and creativity” over perfectionism, or they should leave these sentences out.

AP Lang Argument Essay Example—More Resources

Looking for more tips to help you master your AP Lang Argumentative Essay? Brush up on 20 Rhetorical Devices High School Students Should Know and read our Tips for Improving Reading Comprehension .

If you’re ready to start studying for another part of the AP English Exam, find more expert tips in our How to Write the AP Lang Synthesis and How to Write the AP Lang Rhetorical Essay blog posts.

  • High School Success

Christina Wood

Christina Wood holds a BA in Literature & Writing from UC San Diego, an MFA in Creative Writing from Washington University in St. Louis, and is currently a Doctoral Candidate in English at the University of Georgia, where she teaches creative writing and first-year composition courses. Christina has published fiction and nonfiction in numerous publications, including The Paris Review , McSweeney’s , Granta , Virginia Quarterly Review , The Sewanee Review , Mississippi Review , and Puerto del Sol , among others. Her story “The Astronaut” won the 2018 Shirley Jackson Award for short fiction and received a “Distinguished Stories” mention in the 2019 Best American Short Stories anthology.

  • 2-Year Colleges
  • ADHD/LD/Autism/Executive Functioning
  • Application Strategies
  • Best Colleges by Major
  • Best Colleges by State
  • Big Picture
  • Career & Personality Assessment
  • College Essay
  • College Search/Knowledge
  • College Success
  • Costs & Financial Aid
  • Data Visualizations
  • Dental School Admissions
  • Extracurricular Activities
  • General Knowledge
  • Graduate School Admissions
  • High Schools
  • Homeschool Resources
  • Law School Admissions
  • Medical School Admissions
  • Navigating the Admissions Process
  • Online Learning
  • Outdoor Adventure
  • Private High School Spotlight
  • Research Programs
  • Summer Program Spotlight
  • Summer Programs
  • Teacher Tools
  • Test Prep Provider Spotlight

“Innovative and invaluable…use this book as your college lifeline.”

— Lynn O'Shaughnessy

Nationally Recognized College Expert

College Planning in Your Inbox

Join our information-packed monthly newsletter.

ap lang sample argument essays

Crafting an Impressive Argumentative Essay for AP Lang

ap lang sample argument essays

Writing an argumentative essay for AP Language and Composition requires a strategic approach to effectively convey your perspective. Here's a guide to crafting an impressive argumentative essay:

1. Understand the Prompt:

   - Tip: Carefully read and analyze the prompt. Identify the key elements, including the task you are asked to perform and any specific requirements or constraints.

2. Develop a Clear Thesis Statement:

   - Tip: Formulate a concise and focused thesis statement that clearly states your argument or position. This statement should guide the reader on what to expect in your essay.

3. Identify Your Audience:

   - Tip: Consider your target audience and tailor your language and arguments accordingly. Understanding your audience helps you establish a connection and effectively convey your message.

4. Organize Your Essay Effectively:

   - Tip: Structure your essay with a clear introduction, body paragraphs, and a conclusion. Ensure a logical flow of ideas, with each paragraph contributing to the overall argument.

5. Provide Context and Background:

   - Tip: Begin with a brief introduction that provides context for your argument. Clearly state the issue at hand and offer background information to help readers understand the significance of your argument.

6. Present a Strong Claim:

   - Tip: Clearly articulate your main claim or argument. This should be a debatable statement that forms the core of your essay. Avoid vague or overly broad claims.

7. Support Your Argument with Evidence:

   - Tip: Use relevant and compelling evidence to support your claims. This can include facts, statistics, examples, anecdotes, or quotations. Ensure that your evidence is credible and directly contributes to your argument.

8. Address Counterarguments:

   - Tip: Acknowledge potential counterarguments and address them in your essay. This demonstrates a nuanced understanding of the issue and strengthens your overall argument.

9. Use Persuasive Language:

    - Tip: Choose language that is persuasive and impactful. Utilize rhetorical devices, vivid imagery, and compelling language to engage your readers and convey the urgency or importance of your argument.

10. Craft Well-Reasoned Body Paragraphs:

    - Tip: Dedicate separate paragraphs to different aspects of your argument. Each paragraph should have a clear topic sentence, supporting evidence, and a concise explanation of how the evidence relates to your thesis.

11. Ensure Cohesive Transitions:

    - Tip: Use transitional phrases and sentences to ensure a smooth transition between paragraphs. This enhances the coherence of your essay and guides readers through your line of reasoning.

12. Conclude Effectively:

    - Tip: Summarize your main points in the conclusion and restate the significance of your argument. Avoid introducing new information in the conclusion. End with a strong closing statement that leaves a lasting impression.

13. Revise and Edit:

    - Tip: Set aside time for revision and editing. Review your essay for clarity, coherence, and grammatical accuracy. Ensure that your argument is compelling and well-supported.

14. Seek Feedback:

    - Tip: If possible, seek feedback from peers, teachers, or mentors. External perspectives can provide valuable insights and help you identify areas for improvement.

15. Stay Focused on the Argument:

    - Tip: Throughout the essay, stay focused on your central argument. Avoid unnecessary tangents or diverging from the main point. Every part of your essay should contribute to strengthening your overall argument.

Crafting an impressive argumentative essay for AP Language and Composition involves a combination of persuasive writing techniques, solid evidence, and effective organization. By following these tips, you can create a compelling essay that showcases your ability to engage in sophisticated argumentation and rhetorical analysis.

You Might Also Like

ap lang sample argument essays

Know How to Build a Great College List

Want to choose best college for your study? Get some amazing guidelines that will help you to create a great college list for your admission - Read our blog

ap lang sample argument essays

How to Enhance Your Scholarship Application

Here, you'll get to know all the critical aspects of the college scholarship narratives. We'll also let you know some tips for writing a good narrative.

ap lang sample argument essays

Cracking Admissions to the Most Selective Universities

Want to gain admission to your dream college? Know how can you crack entrance exam to get admissions to the most reputed & selective universities - Read a blog

AP Guru has been helping students since 2010 gain admissions to their dream universities by helping them in their college admissions and SAT and ACT Prep

Free Resources

logo-type-white

AP® English Language

How to get a 6 on the argument frq in ap® english language.

  • The Albert Team
  • Last Updated On: March 1, 2022

How to Get a 6 on the Argument FRQ in AP® English Language

What We Review

Introduction: How to Get a 6 on the Argument FRQ in AP® English Language

Wondering how to get a 6 on the argumentative essay in AP® English Language? 

To score an 5 on the AP® English Argument FRQ question, the CollegeBoard scoring guidelines outline that students need to write an essay that effectively argues a position, uses appropriate and convincing evidence, and showcases a wide range of the elements of writing. Essays that score a 6 do all of that and, additionally, demonstrate sophistication in their argument.

An essay that does all of that is an incredibly well-constructed essay. Such an essay needs a solid framework and excellent support. To do this, it is important to have a clear idea of what you are being asked, to not waffle, to spend time and care with your thesis and outline, and to support every claim you make.

We know the best way to write an AP® English FRQ that does everything right is to understand what you are going to see on the AP® English Language test. Read on to prepare yourself for exam day and earn that 6!

What to Expect from the AP® English Language Argument Free Response Questions

The AP® English argument FRQ is the most straightforward of the AP® English FRQs because it is the most similar to the essays you’re already used to writing. It’s exciting to have free reign and make your own argument, unrestrained from rhetorical analysis devices or documents. But, like most AP® writing, it also can be a little overwhelming.

There’s nothing to read and analyze to provide evidence or help you form an argument. Whether you’re feeling excited or overwhelmed by the AP® writing argument FRQ, consider the rhetorical situation. Be strategic about forming your thesis, craft a strong, chronological argument, and utilize good, supportive evidence to earn a better overall essay response.

Determine the question.

The first question to ask yourself is what am I being asked to do ? This may seem obvious, but it’s surprising how tricky it can be to figure out. Look for keywords and phrases that will answer that question.

Here’s an example from the 2019 AP® English Language argumentative essay.

What to Expect from the AP® English Language Argument Free Response Questions - Determine the Question

Though there are just two short paragraphs, there is a lot of room for confusion here. In this case, “Then, write a well-developed essay in which you explain your judgment.” is the key sentence you are looking for. In 2019, AP® English Language test takers were asked to select a concept, place, role, etc. that they believed was “overrated,” and explain why.

If you cannot determine what the question is, go back and reread the prompt. Focus on the last few sentences, as that’s where you’ll usually find it.

Knowing the question you are answering is the most important part of AP® writing. You will not be able to answer the question effectively if you aren’t certain what the question is. Pick out a specific sentence or two to determine the question, and thereby ensure that you aren’t just writing an essay that responds to the general sense of the argument essay prompts

Pick an opinion and stick to it.

The next step is both simple and difficult. Identify your own opinion on the subject.

But remember — the AP® argumentative essay exam format is designed to test how well you can craft an argument. Questions like the 2019 question seem so daunting, because claiming anything to be “overrated” is such a broad topic. It is a bigger question than students are used to encountering on an AP® test.

But, always remember, there is no right or wrong answer for this AP® English FRQ. And whatever argument you choose will not come back later in the exam or in your final grade in the class. This is not to say that you shouldn’t believe in what you are writing. Only that you should remember that both sides are arguable, pick one, and stick to it. Don’t waffle.

Below we break down two sample student answers from this same 2019 prompt. 

What to Expect from the AP® English Language Argument Free Response Questions - PIck an opinion negative example

In this AP® Lang argument essay example, the student jumps from describing places, to people, to outfits. The prompts asked for only one example and the student gives three.  By doing this, it shows they were not only unable to grasp what the prompt was asking, but that they couldn’t stick to their opinion.  Instead of deeply strengthening one choice, the student gives vague, half-reasons for too many choices. When writing your FRQs, choose just one concept and stick to it.

The following example demonstrates a strong student response:

What to Expect from the AP® English Language Argument Free Response Questions - Pick an opinion strong example

This student picks one clear concept, capitalism, and clearly outlines their support for it.  They write with clear language that opens the door for the deeper analysis coming later in the essay.

Like this student, choose just one clear argument to delve into when writing your FRQ.

Craft a thesis statement.

The thesis statement should be both simple and elegant. Students often find it one of the more difficult writing skills to master, but we’re here to help. Just remember that it should encompass your entire essay in just one sentence.  So, for the 2019 argument FRQ :

Good thesis: While capitalism undeniably has its upsides, it has many downsides that are rarely recognized. When considering the downsides, capitalism is clearly overrated as it commodifies humanity and uplifts a minority at the expense of the majority.

This thesis breaks down a) that the author clearly states his claim that capitalism is overrated, b) that the author will support that claim with examples on how it commodifies humanity and how it hurts the majority in favor of the minority.

Good thesis: While the Electoral College was created in the name of equality for smaller states, it is ultimately overrated because it undercuts the popular vote, it is an archaic practice that is unsuitable for the modern era.

This thesis claims the Electoral College is overrated by claiming it doesn’t do what it was created to do in the first place- support equality.  It also introduces two supporting examples for the rest of the essay- it undercuts the popular vote and it doesn’t work in the modern era.

Not a good thesis: Kicking a ball in a net and scoring, is not as important as saving lives. Soccer to me would be considered overrated.

This thesis doesn’t give clear direction for the rest of the essay.  The author claims soccer is overrated, but doesn’t tell us why. The example that “it’s not as important as saving lives” is unrelated and also not touched on again later in the essay. This thesis isn’t specific and doesn’t give you a clear idea of what the author will be saying next.

Not a good thesis: The term “overrated” has been used in conversation to diminish the value of roles. In unusual circumstances the term “overrated” should be applied to the idea of freedom in regards to social change, but overall it should not be applied in regards to global devastation and cruel treatment.

This thesis does not directly answer the question.  Is the author arguing that freedom is overrated? They also claim that the term overrated doesn’t apply to global devastation and cruel treatment. This second claim is both unrelated to the first and doesn’t work to answer the initial prompt.

Looking at these four examples, can you see the difference between a strong and weak thesis?

After you’ve determined your thesis, use it as a jumping point to sketch a quick outline. Then, follow your outline, bringing in your own concrete examples and evidence. Doing so will improve your AP® writing.

Return to the Table of Contents

Craft a chronological argument.

A good argument builds as you move through the essay. It does not simply repeat the same points. Instead, the different points of the argument build off one another and work together to advance the author’s point.

Let’s look at the 2018 AP® English argument FRQ for an example.

What to Expect from the AP® English Language Argument Free Response Questions - Craft a chronological argument

In this case, students are being asked to argue a position on the value of choosing the unknown. 

All students are likely to have their own definitions of what “choosing the unknown” might mean. You first want to consider what this phrase means to you, and how it applies to the real world.  Could it mean breaking out of your comfort zone in daily routines, or could it mean going to theater school to follow your dreams?  There’s no wrong answers, but try to pin down one. Consider Lindbergh’s quote the prompt gives you, and how shock, disappointment, and enrichment play into choosing the unknown.

Once you’ve nailed down your definition, you can begin to form your arguments. A chronological argument builds off itself. So, in this question’s case, an outline would look something like this:

  • Choosing the unknown is necessary for the development of the human race.
  • Scientific advancements cannot be made without testing the boundaries of the unknown.
  • Cultural and artistic growth can only occur through exploring the unknown.

First, a student must define what choosing the unknown means, and what makes it difficult. Next the student argues for the value of choosing the unknown, in that the human race could never develop without it.  Finally, the student will argue for the invaluable scientific and cultural/artistic advances made throughout history by breaking known boundaries.

When you sketch your outline, quickly ask yourself if the outline would make just as much sense if you rearranged it. If the answer is no, start writing your essay. If the answer is yes, try to structure your argument so that your points build off one another.

Support your claims.

All arguments need evidence. This is the proof you need to support your thesis. And in the case of the AP® English argument FRQ, the evidence all comes from you. What exactly that evidence is will vary from question to question and from student to student. But make sure that every point you make is supported by evidence.

Here’s some good news — you already know quite a bit about effective evidence from what you have learned in AP® English about rhetorical devices. Your main purpose in this essay is to persuade. What have you learned in class about effective ways to persuade? What rhetorical devices can you utilize? Try to pick the best devices to support your argument that you can.

Here are some examples of supportive and non-supportive evidence that students could use to support their claims.

What to Expect from the AP® English Language Argument Free Response Questions - Support your claims

The 2017 AP® English language argument FRQ asked students to argue a position if the most essential skill is artifice. The example student answers given below are from here .

Supportive evidence:   “Throughout history, rulers have utilized countless different methods of achieving power, however none have been so successful as mastering the art of lying.

In his advice to future rulers, Niccolo Machiavelli encouraged them to lie and maintain the illusion of sympathy to the common struggles in order to retain power. He asserts that it is imperative for a ruler to appear caring and sympathetic even if he has no objective but power.

Machiavelli argues that to be sincere and honest is akin to being vulnerable. A ruler must be skilled in the art of deception if he is not to fall prey to usurpers. Thus, it is essential that he appear humble and morally upright to his constituents as he is to appear idealistic, despite his nature being identical to his citizens.”

In this paragraph, the student chooses to discuss the role of artifice in politics. The student claims that mastering lying is essential to achieving political power. The student uses Machiavelli’s leadership and beliefs as specific examples to support this, by analyzing and connecting each point back to his/her claim.

Non-supportive evidence: “Another example would be actors on red carpets or at interviews they sound generous and relatable, but in reality they could be selfish people who don’t care about anyone. To the public they act charming, honest, and sincere. They do this so they can get famous and rich. They do this so they will never get ignored.”

In this paragraph, the student chooses to discuss the role of artifice in the culture of entertainment and celebrities. However, the student does not utilize supportive evidence to do so. The paragraph is full of claims about how actors lie, but does not provide a concrete example to anchor the claims. The student provides a lot of very vague generalizations, but no clear evidence or examples of specific celebrities and how they used artifice to succeed.

There is so much variance in prompts and students’ prior knowledge; it’s impossible to provide a checklist of what makes evidence supportive. But a good trick to decide if you’ve supported your claims well enough is to talk to yourself. No really, it’s a good idea.

Picture yourself discussing your essay with someone. Imagine that this person disagrees with everything that you say. Every time you make a claim, like that it’s important to be polite in an email, your imaginary person shakes their head and tells you no. How would you try to convince them? What examples would you use? Make sure that for each opinion you put forward; you have provided an answer to someone who would disagree with you.

The evidence is an important part of your essay. If your outline and your argument are a framework, your evidence is the brick and mortar. A house without brick and mortar won’t fall, but it won’t be a very nice house to inhabit. Tie every claim you make to a piece of evidence to ensure the best essay possible.

Wrapping Things Up: Scoring a 6 on the Argument FRQ for AP® English Language

The AP® English argument FRQ varies quite a bit. But it is ultimately about how well you can put forth an argument. So, don’t be afraid to spend some time crafting that argument. We’ve covered a lot in this article- here are the main points to remember:

  • Determine the question. Figure out what the prompt is asking you to do.
  • Pick an opinion and stick to it. Choose one side of the argument and one clear claim to support all the way through.
  • Craft a thesis statement. Your thesis should be clear, concise, and introduce the content of your essay.
  • Craft a chronological argument. Make an argument that builds on its prior points.
  • Support your claims. Support yourself with concrete, specific evidence and examples. 

But most of all, have fun. This essay is the one you should be looking forward to, where you have the freest rein. Enjoy it and earn yourself a 6.

Do the examples shown make sense to you? Can you picture yourself moving through the AP® writing argument FRQ with ease now?

Interested in a school license?​

8 thoughts on “how to get a 6 on the argument frq in ap® english language”.

Thank you for explaining this so eloquently. Excellent post, I will keep this handy and refer to it often from now on. It’s so educative. Great post!

Sure, glad it helped.

I’m an AP® Language teacher and the title of your article caught my eye because the essays aren’t scored on a 0-9 scale anymore. The max score for an essay now is a 6. Essays are now scored in 3 categories: Thesis: 0 or 1 point Evidence and commentary: 0-4 points Sophistication: 0 or 1 point I just wanted to let you know! I saw this was last updated in 2020 and just thought it should reflect the current AP® exam.

Thank you for the heads up! This is an older blog post that must have had something else updated to it this year. We’ve gone ahead and revised the post.

Hi, my AP® Language teacher emphasized on a counterargument at the end of the supporting paragraphs. Could you elaborate on it? Also, how exactly do we get the sophistication point?

Hi Stephanie, thanks for reaching out! Making a solid counter-argument is definitely one way to make sure that you earn the Sophistication point. We recommend having a look at our AP® English Language Review Guide for 2021 for more tips! The College Board’s Free-Response Question and Scoring Information Archive also provides authentic examples of student writing — many of which successfully make counterarguments and rebuttals to earn the Sophistication point.

Hi can I get a 6?

Hi Roy, we certainly believe that earning a 6 on your FRQs is possible with practice and dedication! I’d recommend having a look at our AP® English Language Review Guide for tips and tricks, and you can also browse our AP® English Language and Composition Resource Page and Free Response practice questions for targeted practice.

Comments are closed.

Popular Posts

AP® Physics I score calculator

AP® Score Calculators

Simulate how different MCQ and FRQ scores translate into AP® scores

ap lang sample argument essays

AP® Review Guides

The ultimate review guides for AP® subjects to help you plan and structure your prep.

ap lang sample argument essays

Core Subject Review Guides

Review the most important topics in Physics and Algebra 1 .

ap lang sample argument essays

SAT® Score Calculator

See how scores on each section impacts your overall SAT® score

ap lang sample argument essays

ACT® Score Calculator

See how scores on each section impacts your overall ACT® score

ap lang sample argument essays

Grammar Review Hub

Comprehensive review of grammar skills

ap lang sample argument essays

AP® Posters

Download updated posters summarizing the main topics and structure for each AP® exam.

All Subjects

Argument Essay: Evidence

8 min read • june 18, 2024

Stephanie Kirk

Stephanie Kirk

We aren’t sure where it started, but many teachers use REHUGO to help students find evidence on the Argument FRQ .  This acronym provides a quick check that can help you build logical evidence that supports your claim .

  • R - Reading - Something you have read, fiction or nonfiction, that connects the given topic.
  • E - Entertainment - A movie or song with dialogue or lyrics that present related ideas.
  • H - History - An event, document, speech, or person from history that aligns with the given topic.
  • U - Universal Truths - A common maxim or socially-accepted quote people tend to accept as truth.
  • G - Government - A national or international current event or governmental situation related to the topic.
  • O - Observations - Any cultural, technical, or societal trend that relates to the topic.

Suggested Guided Questions for the Argument FRQ

Now that you have a better understanding of the Argument FRQ’s expectations and scoring, let’s visit a sample prompt and add a few guided questions that you can use to help plan your own writings.

In his book   Canadian journalist Malcolm Gladwell (born 1963) writes: “To assume the best about another is the trait that has created modern society. Those occasions when our trusting nature gets violated are tragic. But the alternative—to abandon trust as a defense against predation and deception—is worse.”Write an essay that argues your position on the importance of .

Guided Question 1: What does the prompt say? 📝

  • Why do I do this?  Understanding the concept or idea presented by the prompt is vital to planning a response that thoroughly addresses the prompt and stays on topic throughout.  This is where you are going to  BAT the PROMPT .
  • Background : Gladwell asserts that society should trust each other in order to continue to be productive. Assuming the best about each other presents a better outcome than assuming the worst about each other.
  • Advice : The new stable prompt wording does not give much advice, but you should revisit advice you learned in class or from us as Fiveable -- things like using Toulmin to plan your response and planning modes of development that help progress your reasoning.
  • Task : Write an essay giving your position about the importance of trust. Specifically, is Gladwell right or wrong? And why? 🎥 Watch: AP Lang -   Argumentation, Part I: It's a Trap!

Rhetorical Situation : When writing for AP Lang, it is important to consider the rhetorical situation and write in a manner that demonstrates an understanding of all elements of that situation. 

  • Context - the historical, social, and cultural movements in the time of the text
  • Exigence - the urgency that leads to an action
  • Purpose - the goal the speaker wants to achieve and the desired audience movement
  • Persona - the “mask” shown to his/her audience

ap lang sample argument essays

Guided Question 2: What do I think? 💡

Why do I do this?   Taking a moment to brainstorm ideas can help organize thoughts and build an outline that you can revisit if you lose your train of thought in the stress of timed writing.

What does it look like?   This might just be stream-of-consciousness in your head, cloud diagrams, or even bulleted notes on the side of your prompt, but it needs to end with a clear position statement you can use for your thesis statement . For example: Trust is important. It does suck to get betrayed though but having a positive outlook creates positive results. Thinking the worst makes people act negatively because they project in a way that leads toward the worst response. ⬇️

  • Thesis Statement: Although some people believe humanity seems self-interested, a trusting nature enables individuals to focus on the positive and treat others with the respect that foster positive interrelationships.

Guided Question 3: What evidence can I use? 🤔

  • Why do I do this?   Revisit REHUGO and use Toulmin to plan your body paragraphs based on the thesis statement you came to when brainstorming ideas.
  • Modes of Development:  When writing, it is helpful to arrange the overall essay and its parts in a way that aligns with the purpose.  Consider these basic modes and how you might use them in writing an argumentative essay.- Cause and Effect attempt to follow the chain of events and establish causation. The description brings imagery and details into a text so that it set up the tone and ensures the reader can follow the mood.- Classification allows the speaker to categorize things in a way that guides the reader to follow the line of reasoning.- Comparison , looking at the similarities and differences, helps to analyze the intricate details of a given topic.  Because this describes differing elements, it may be structured by the element or by the characteristic.- Definition  works to explain what something is or is not. By defining the subject being discussed, the speaker is able to control the thinking about that subject.  Because this helps to clarify the topic, it is generally used in the introductory section of argumentation.- Exemplification  is used when explaining the topic or situation by giving examples to help lead the audience to the desired conclusion.- Narration  tells a story or gives an anecdote to help illustrate the point.- Process Analysis serves to explain the process by which something is done.
  • What does it look like?   I always use a version of the T-chart because one side is my evidence and the other side helps me consider multiple perspectives.  You may not have an idea in all areas, and you may have multiple ideas in one area. Try to time yourself so that you get plenty of strong evidence without wasting too much time. | Supports 🏗️ | Thoughts 💭 | | --- | --- | | ~ R -  Trust in Society by Cook points out that we only realize the importance of trust when there’s a breakdown.~ R - Essay “Importance of Trust” from class said that trust is not easy but it is what builds the foundation of a relationship and drives all actions between sides.~ E -  The 2011 movie  Trust shows that too much trust can get you hurt or killed.~ E - The media has so much bias it can’t be trusted… ~ H - Revolutionary War - trusted founding fathers and God’s position resulting in breaking free from Britain~ U - Trust - People should be trusted until proven otherwise~ G - A criminal escaped in our town and was on the loose; we trusted police to do their job rather than resort to vigilante justice~ G - Trust the president will have our best interests at heart -- most appear to have done that~ O - App-based dating - relies on trusting the person you meet online before meeting in person  | > relationships between couples> trust in contractual agreements? Moral obligation for trust?> Counter?> Relied on trust and won with not much else to go ob> Ex, Parents, Teachers, Siblings > Trust helped keep us safe; job was well done when we were out of the way> Obvious exceptions; System of impeachment  |

🎥 Watch: AP Lang -   Review: Argument Body Paragraphs

PRO TIP:  What if you have more evidence for the other side? Well, you haven’t started writing just yet, so it isn’t too late to go back and revise the thesis statement. Sometimes this happens in looking for evidence, and that’s perfectly okay.  In fact, many times students will say they wrote an essay using evidence that went completely against what they felt in their head or heart just because they couldn’t put a logical argument on paper without getting too emotional.   Know your audience, and build your argument.

Guided Question 4: How should I effectively organize my response? 📈

  • Why do I do this?   This step helps to outline the response so that your ideas are organized before you start writing.  The general advice is to follow the structure of  Classical Argumentation , but there is no rule saying that must be done to score well on the rubric. 💯
  • What does it look like?   One way of doing this would be to mark numbers by ideas tracked and written in the brainstorm, but some do take a moment to build an outline with topic sentences.
  • Modes of Development:  When writing, it is helpful to arrange the overall essay and its parts in a way that aligns with the purpose.  Consider these basic modes and how you might use them in writing an argumentative essay.- Cause and Effect attempt to follow the chain of events and establish causation. The description brings imagery and details into a text so that it set up the tone and ensures the reader can follow the mood.- Classification allows the speaker to categorize things in a way that guides the reader to follow the line of reasoning.- Comparison , looking at the similarities and differences, helps to analyze the intricate details of a given topic.  Because this describes differing elements, it may be structured by the element or by the characteristic.- Definition works to explain what something is or is not. By defining the subject being discussed, the speaker is able to control the thinking about that subject.  Because this helps to clarify the topic, it is generally used in the introductory section of argumentation.- Exemplification is used when explaining the topic or situation by giving examples to help lead the audience to the desired conclusion.- Narration tells a story or gives an anecdote to help illustrate the point.- Process Analysis serves to explain the process by which something is done.
  • Start by creating  Toulmin Position Statements that can be used for topic sentences and then move into a writing plan. Here’s a sample for this prompt:

Sample Outline:

  • Revised Thesis: Although some people believe trust must be earned, maintaining a trusting nature is important because it enables society to focus on positivity and create positive interrelationships that lead to positive outcomes.
  • Universal truth
  • Observation
  • Entertainment - refute
  • Conclude: When considering the value of trust in society, it is clear that the benefits of granting trust far outweigh the consequences of withholding it.

Key Terms to Review ( 35 )

© 2024 fiveable inc. all rights reserved., ap® and sat® are trademarks registered by the college board, which is not affiliated with, and does not endorse this website..

PrepScholar

Choose Your Test

  • Search Blogs By Category
  • College Admissions
  • AP and IB Exams
  • GPA and Coursework

Expert Guide to the AP Language and Composition Exam

author image

Advanced Placement (AP)

article-71342_640.jpg

With the 2023 AP English Language and Composition exam happening on Tuesday, May 9, it's time to make sure that you're familiar with all aspects of the exam. In this article, I'll give a brief overview of the test, do a deeper dive on each of the sections, discuss how the exam is scored, offer some strategies for studying, and finally wrap up with some essential exam day tips.

Exam Overview

The AP Language and Composition exam tests your rhetorical and composition skills. Essentially, how do authors construct effective arguments in their writing? What tools do they use? How can you use those tools to craft effective writing yourself? That is the essence of rhetorical analysis.

The exam has two parts: the first section is an hour-long, 45 question multiple-choice section. It includes five sets of questions, each based on a passage or passages. In this section, there will be 23-25 rhetorical analysis questions which test your rhetorical skills. There will also be 20-22 writing questions which require you to consider revisions to the texts you're shown.

The second section is free response. It starts with a 15-minute reading period, and then you'll have 120 minutes to write three analytical essays:

  • One essay where you synthesize several provided texts to create an argument
  • One essay where you analyze a nonfiction passage for its rhetorical construction
  • One essay where you create an original argument in response to a prompt.

You will have about 40 minutes to write each essay, but no one will prompt you to move from essay to essay—you can structure the 120 minutes as you wish.

In the next sections I'll go over each section of the exam more closely—first multiple choice, and then free response.

The AP English Language and Composition Multiple-Choice

The multiple-choice section tests you on two main areas. The first is how well you can read and understand nonfiction passages for their use of rhetorical devices and tools. The second is how well you can "think like a writer" and make revisions to texts in composition questions.

You will be presented with five passages, about which you will receive a small amount of orienting information, e.g. "This passage is excerpted from a collection of essays on boating" or "This passage is excerpted from an essay written in 19th-century Haiti." Each passage will be followed by a set of questions.

There are, in general, eight question types you can expect to encounter on the multiple-choice section of the exam. I've taken my examples from the sample questions in the " Course and Exam Description ."

eight-1316133_640.jpg

Magic eight-ball says there are eight types of multiple-choice questions!

Type 1: Reading Comprehension

These questions are focused on verifying that you understood what a certain part of the passage was saying on a concrete, literal level. You can identify these questions from phrases like "according to" "refers," etc. The best way to succeed on these questions is to go back and re-read the part of the passage referred to very carefully.

Comprehension.png

Type 2: Implication

These questions take reading comprehension one step further—they are primarily focused on what the author is implying without directly coming out and saying it. These questions will have a correct answer, though, based on evidence from the passage. Which interpretation offered in the answers does the passage most support? You can identify questions like these from words like "best supported," ‘"implies," "suggests," "inferred," and so on.

implies.png

Type 3: Overall Passage and Author Questions

These questions ask about overall elements of the passage or the author, such as the author's attitude on the issue discussed, the purpose of the passage, the passage's overarching style, the audience for the passage, and so on.

You can identify these questions because they won't refer back to a specific moment in the text. For these questions, you'll need to think of the passage from a "bird's-eye view" and consider what all of the small details together are combining to say.

3overall_passage.png

Type 4: Relationships Between Parts of the Text

Some questions will ask you to describe the relationship between two parts of the text, whether they are paragraphs or specific lines. You can identify these because they will usually explicitly ask about the relationship between two identified parts of the text, although sometimes they will instead ask about a relationship implicitly, by saying something like "compared to the rest of the passage."

4relationship.png

Type 5: Interpretation of Imagery/Figurative Language

These questions will ask you about the deeper meaning or implication of figurative language or imagery that is used in the text. Essentially, why did the author choose to use this simile or this metaphor? What is s/he trying to accomplish?

You can generally identify questions like this because the question will specifically reference a moment of figurative language in the text. However, it might not be immediately apparent that the phrase being referenced is figurative, so you may need to go back and look at it in the passage to be sure of what kind of question you are facing.

5imagery.png

Type 6: Purpose of Part of the Text

Still other questions will ask you to identify what purpose a particular part of the text serves in the author's larger argument. What is the author trying to accomplish with the particular moment in the text identified in the question?

You can identify these questions because they will generally explicitly ask what purpose a certain part of the text serves. You may also see words or phrases like "serves to" or "function."

6purpose_of_part.png

Type 7: Rhetorical Strategy

These questions will ask you to identify a rhetorical strategy used by the author. They will often specifically use the phrase "rhetorical strategy," although sometimes you will be able to identify them instead through the answer choices, which offer different rhetorical strategies as possibilities.

7rhetorical_strategy.png

Type 8: Composition

This is the newest question type, first seen in the 2019/2020 school year. For these questions, the student will need to act as though they are the writer and think through different choices writers need to make when writing or revising text.

These questions can involve changing the order of sentences or paragraphs, adding or omitting information to strengthen an argument or improve clarity, making changes to draw reader attention, and other composition-based choices.

body_composition

Some very important stylish effects going on here.

The AP English Language and Composition Free Response

The free response section has a 15-minute reading period. After that time, you will have 120 minutes to write three essays that address three distinct tasks.

Because the first essay involves reading sources, it is suggested that you use the entire 15-minute reading period to read the sources and plan the first essay. However, you may want to glance at the other questions during the reading period so that ideas can percolate in the back of your mind as you work on the first essay.

Essay One: Synthesis

For this essay, you will be briefly oriented on an issue and then given anywhere from six to seven sources that provide various perspectives and information on the issue. You will then need to write an argumentative essay with support from the documents.

If this sounds a lot like a DBQ , as on the history AP exams, that's because it is! However, this essay is much more argumentative in nature—your goal is to persuade, not merely interpret the documents.

Example (documents not included, see 2022 free response questions ):

body-AP-Literature-synthesis

Essay Two: Rhetorical Analysis

In the second essay, you'll be presented with an excerpt from a nonfiction piece that advances an argument and asked to write an essay analyzing the rhetorical strategies used to construct the passage's argument. You will also be given some orienting information—where the passage was excerpted from, who wrote it, its approximate date, where it was published (if at all), and to whom it was directed.

Example (excerpt not included, see 2022 free response questions ):

body-AP-literature-Question-2

Essay Three: Argument

In the third essay, you will be presented with an issue and asked to write a persuasive essay taking a position on the issue. You will need to support your position with evidence from your "reading, experience, and observations."

body-AP-lit-Question-3

This doesn't look like a very well-constructed argument.

How The AP Language and Composition Exam Is Scored

The multiple-choice section of the exam is worth 45% of your score, and the free-response section is worth the other 55%. So each of the three free-response essays is worth about 18% of your score.

As on other APs, your raw score will be converted to a scaled score of 1-5. This exam has a relatively low 5 rate. Only 10% of test takers received a 5 in 2022 , although 56% of students received a score of 3 or higher.

In terms of how the raw score is obtained, the multiple-choice section is similar to other AP multiple-choice sections: you receive a point for every question you answer correctly, and there is no penalty for guessing.

The grading rubrics for the free-response questions were revamped in 2019. They are scored using analytic rubrics instead of holistic rubrics. For each free-response question, you will be given a score from 0-6. The rubrics assess three major areas:

#1: Thesis (0 to 1 points): Is there a thesis, and does it properly respond to the prompt?

#2: Evidence and Commentary (0 to 4 points): Does the essay include supporting evidence and analysis that is relevant, specific, well organized, and supports the thesis?

#3: Sophistication (0 to 1 points): Is the essay well-crafted and does it show a sufficiently nuanced understanding of the prompt?

Each scoring rubric broadly assesses these three factors. However, each task is also different in nature, so the rubrics do have some differences. I'll go over each rubric—and what it really means—for you here.

Synthesis Essay Rubrics

0 For any of the following:
1

EVIDENCE AND COMMENTARY

0
1 AND
2 AND
3 AND
4 AND

SOPHISTICATION

0
1 Responses that earn this point may demonstrate sophistication of thought and/or a complex understanding of the rhetorical situation by doing any of the following:

dough-196235_640.jpg

Time to synthesize this dough into some cookies.

Rhetorical Analysis Essay Rubrics

0
1 AND
2 AND
3 AND AND
4 AND AND

magnifying-glass-914922_640.png

Examine your texts closely!

Argumentative Essay Rubrics

playing-puppies-790638_640.jpg

The best kind of frenzy is a puppy frenzy!

AP English Language Prep Tips

Unlike its cousin, the AP English Literature and Composition exam, the AP Language and Composition exam (and course) have very little to do with fiction or poetry. So some students used to more traditional English classes may be somewhat at a loss as to what to do to prepare.

Luckily for you, I have a whole slate of preparation tips for you!

Read Nonfiction—In a Smart Way

A major thing you can do to prepare for the AP Lang and Comp exam is to read nonfiction— particularly nonfiction that argues a position , whether explicitly (like an op-ed) or implicitly (like many memoirs and personal essays). Read a variety of non-fiction genres and topics, and pay attention to the following:

  • What is the author's argument?
  • What evidence do they use to support their position?
  • What rhetorical techniques and strategies do they use to build their argument?
  • Are they persuasive? What counterarguments can you identify? Do they address them?

Thinking about these questions with all the reading you do will help you hone your rhetorical analysis skills.

Learn Rhetorical Terms and Strategies

Of course, if you're going to be analyzing the nonfiction works you read for their rhetorical techniques and strategies, you need to know what those are! You should learn a robust stable of rhetorical terms from your teacher, but here's my guide to the most important AP Language and Composition terms .

  • We've compiled a list of 20 rhetorical devices you should know.
  • A heroic individual from Riverside schools in Ohio uploaded this aggressively comprehensive list of rhetorical terms with examples. It's 27 pages long, and you definitely shouldn't expect to know all of these for the exam, but it's a useful resource for learning some new terms.
  • Another great resource for learning about rhetorical analysis and how rhetorical devices are actually used is the YouTube Channel Teach Argument , which has videos rhetorically analyzing everything from Taylor Swift music videos to Super Bowl commercials. It's a fun way to think about rhetorical devices and get familiar with argumentative structures.
  • Finally, a great book—which you might already use in your class—is " They Say, I Say. " This book provides an overview of rhetoric specifically for academic purposes, which will serve you well for AP preparation and beyond.

You also need to practice argumentative and persuasive writing. In particular, you should practice the writing styles that will be tested on the exam: synthesizing your own argument based on multiple outside sources, rhetorically analyzing another piece of writing in-depth, and creating a completely original argument based on your own evidence and experience.

You should be doing lots of writing assignments in your AP class to prepare, but thoughtful, additional writing will help. You don't necessarily need to turn all of the practice writing you do into polished pieces, either—just writing for yourself, while trying to address some of these tasks, will give you a low-pressure way to try out different rhetorical structures and argumentative moves, as well as practicing things like organization and developing your own writing style.

once-upon-a-time-719174_640.jpg

Not the most auspicious start to an argumentative essay.

Practice for the Exam

Finally, you'll need to practice specifically for the exam format. There are sample multiple-choice questions in the " AP Course and Exam Description ," and old free-response questions on the College Board website.

Unfortunately, the College Board hasn't officially released any complete exams from previous years for the AP English Language and Composition exam, but you might be able to find some that teachers have uploaded to school websites and so on by Googling "AP Language complete released exams." I also have a guide to AP Language and Composition practice tests .

Once you're prepped and ready to go, how can you do your best on the test?

Looking for help studying for your AP exam? Our one-on-one online AP tutoring services can help you prepare for your AP exams. Get matched with a top tutor who got a high score on the exam you're studying for!

AP Language and Composition Test Day Tips

Here are four key tips for test-day success.

board-1193334_640.jpg

You are one hundred percent success!

Interact With the Text

When you are reading passages, both on the multiple-choice section and for the first two free-response questions, interact with the text! Mark it up for things that seem important, devices you notice, the author's argument, and anything else that seems important to the rhetorical construction of the text. This will help you engage with the text and make it easier to answer questions or write an essay about the passage.

Think About Every Text's Overarching Purpose and Argument

Similarly, with every passage you read, consider the author's overarching purpose and argument. If you can confidently figure out what the author's primary assertion is, it will be easier to trace how all of the other aspects of the text play into the author's main point.

Plan Your Essays

The single most important thing you can do for yourself on the free-response section of the AP English Language exam is to spend a few minutes planning and outlining your essays before you start to write them.

Unlike on some other exams, where the content is the most important aspect of the essay, on the AP Language Exam, organization, a well-developed argument, and strong evidence are all critical to strong essay scores. An outline will help you with all of these things. You'll be able to make sure each part of your argument is logical, has sufficient evidence, and that your paragraphs are arranged in a way that is clear and flows well.

Anticipate and Address Counterarguments

Another thing you can do to give your free responses an extra boost is to identify counterarguments to your position and address them within your essay. This not only helps shore up your own position, but it's also a fairly sophisticated move in a timed essay that will win you kudos with AP graders.

envelope-392962_640.jpg

Address counterarguments properly or they might get returned to sender!

Key Takeaways

The AP Language and Composition exam tests your rhetorical skills. The exam has two sections.

The first section is an hour-long, 45 question multiple-choice test based on the rhetorical techniques and composition choices.

The second section is a two-hour free-response section (with a 15-minute initial reading period) with three essay questions: one where you must synthesize given sources to make an original argument, one where you must rhetorically analyze a given passage, and one where you must create a wholly original argument about an issue with no outside sources given.

You'll receive one point for every correct answer on the multiple-choice section of the exam, which is worth 45% of your score. The free-response section is worth 55% of your score. For each free-response question, you'll get a score based on a rubric from 0-6. Your total raw score will be converted to a scaled score from 1-5.

Here are some test prep strategies for AP Lang:

#1 : Read nonfiction with an eye for rhetoric #2 : Learn rhetorical strategies and techniques #3 : Practice writing to deploy rhetorical skills #4 : Practice for the exam!

Here are some test-day success tips:

#1 : Interact with each passage you encounter! #2 : Consider every text's overarching purpose and argument. #3 : Keep track of time #4 : Plan your essays #5 : Identify and address counterarguments in your essays.

With all of this knowledge, you're ready to slay the AP English Language and Composition beast!

animal-1299749_640.png

Noble knight, prepare to slay the AP dragon!

What's Next?

Want more AP Lang review? We have a complete collection of released AP Language practice tests , as well as a list of the AP Lang terms you need to know and a guide to the multiple choice section .

Taking the AP Literature exam? Check out our ultimate guide to the AP English Literature test and our list of AP Literature practice tests .

Taking other AP exams? See our Ultimate Guides to AP World History , AP US History , AP Chemistry , AP Biology , AP World History , and AP Human Geography .

Need more AP prep guidance? Check out how to study for AP exams and how to find AP practice tests .

Want to build the best possible college application?   We can help.   PrepScholar Admissions combines world-class admissions counselors with our data-driven, proprietary admissions strategies. We've guided thousands of students to get into their top choice schools, from state colleges to the Ivy League. We know what kinds of students colleges want to admit and are driven to get you admitted to your dream schools. Learn more about PrepScholar Admissions to maximize your chance of getting in:

These recommendations are based solely on our knowledge and experience. If you purchase an item through one of our links, PrepScholar may receive a commission.

Trending Now

How to Get Into Harvard and the Ivy League

How to Get a Perfect 4.0 GPA

How to Write an Amazing College Essay

What Exactly Are Colleges Looking For?

ACT vs. SAT: Which Test Should You Take?

When should you take the SAT or ACT?

Get Your Free

PrepScholar

Find Your Target SAT Score

Free Complete Official SAT Practice Tests

How to Get a Perfect SAT Score, by an Expert Full Scorer

Score 800 on SAT Math

Score 800 on SAT Reading and Writing

How to Improve Your Low SAT Score

Score 600 on SAT Math

Score 600 on SAT Reading and Writing

Find Your Target ACT Score

Complete Official Free ACT Practice Tests

How to Get a Perfect ACT Score, by a 36 Full Scorer

Get a 36 on ACT English

Get a 36 on ACT Math

Get a 36 on ACT Reading

Get a 36 on ACT Science

How to Improve Your Low ACT Score

Get a 24 on ACT English

Get a 24 on ACT Math

Get a 24 on ACT Reading

Get a 24 on ACT Science

Stay Informed

Get the latest articles and test prep tips!

Follow us on Facebook (icon)

Ellen has extensive education mentorship experience and is deeply committed to helping students succeed in all areas of life. She received a BA from Harvard in Folklore and Mythology and is currently pursuing graduate studies at Columbia University.

Ask a Question Below

Have any questions about this article or other topics? Ask below and we'll reply!

What are your chances of acceptance?

Calculate for all schools, your chance of acceptance.

Duke University

Your chancing factors

Extracurriculars.

ap lang sample argument essays

How to Write the AP Lang Synthesis Essay + Example

Do you know how to improve your profile for college applications.

See how your profile ranks among thousands of other students using CollegeVine. Calculate your chances at your dream schools and learn what areas you need to improve right now — it only takes 3 minutes and it's 100% free.

Show me what areas I need to improve

What’s Covered:

What is the ap lang synthesis essay, how will ap scores affect my college chances.

AP English Language and Composition, commonly known as AP Lang, is one of the most engaging and popular AP classes offered at most high schools, with over 535,000 students taking the class . AP Lang tests your ability to analyze written pieces, synthesize information, write rhetorical essays, and create cohesive and concrete arguments. However, the class is rather challenging as only 62% of students were able to score a three or higher on the exam. 

The AP Lang exam has two sections. The first consists of 45 multiple choice questions which need to be completed in an hour. This portion counts for around 45% of your total score. These questions ask students to analyze written pieces and answer questions related to each respective passage.  All possible answer choices can be found within the text, and no prior knowledge of literature is needed to understand the passages.

The second section contains three free-response questions to be finished in under two hours and 15 minutes. This section counts for 55% of your score and includes the synthesis essay, the rhetorical essay, and the argumentative essay.

  • The synthesis essay requires you to read 6-7 sources and create an argument using at least three sources.
  • The rhetorical analysis essay requires you to describe how a piece of writing evokes specific meanings and symbolism.
  • The argumentative essay requires you to pick a perspective of a debate and create an argument based on the evidence provided.

In this post, we will take a look at the AP Lang synthesis essay and discuss tips and tricks to master this part of the exam. We will also provide an example of a well-written essay for review.  

The AP Lang synthesis essay is the first of three essays included in the Free Response section of the AP Lang exam. The exam presents 6-7 sources that are organized around a specific topic, with two of those sources purely visual, including a single quantitative source (like a graph or pie chart). The remaining 4-5 sources are text-based, containing around 500 words each. It’s recommended that students spend an hour on this essay—15 minute reading period, 40 minutes writing, and 5 minutes of spare time to check over work.

Each synthesis essay has a topic that all the sources will relate to. A prompt will explaining the topic and provide some background, although the topics are usually broad so you will probably know something related to the issue. It will also present a claim that students will respond to in an essay format using information from at least three of the provided sources. You will need to take a stance, either agreeing or disagreeing with the position provided in the claim. 

According to the CollegeBoard, they are looking for essays that “combine different perspectives from sources to form a support of a coherent position.” This means that you must state your claim on the topic and highlight relationships between several sources that support your specific position on the topic. Additionally, you’ll need to cite clear evidence from your sources to prove your point.

The synthesis essay counts for six points on the AP Lang exam. Students can receive 0-1 points for writing a thesis statement, 0-4 based on the incorporation of evidence and commentary, and 0-1 points based on the sophistication of thought and demonstration of complex understanding.

While this essay seems extremely overwhelming, considering there are a total of three free-response essays to complete, with proper time management and practiced skills, this essay is manageable and straightforward. In order to enhance the time management aspect of the test to the best of your ability, it is essential to divide the essay up into five key steps.

Step 1: Analyze the Prompt

As soon as the clock starts, carefully read and analyze what the prompt asks from you. It might be helpful to markup the text to identify the most critical details. You should only spend around 2 minutes reading the prompt so you have enough time to read all the sources and figure out your argument. Don’t feel like you need to immediately pick your stance on the claim right after reading the prompt. You should read the sources before you commit to your argument.

Step 2: Read the Sources Carefully

Although you are only required to use 3 of the 6-7 sources provides, make sure you read ALL of the sources. This will allow you to better understand the topic and make the most educated decision of which sources to use in your essay. Since there are a lot of sources to get through, you will need to read quickly and carefully.

Annotating will be your best friend during the reading period. Highlight and mark important concepts or lines from each passage that would be helpful in your essay. Your argument will probably begin forming in your head as you go through the passages, so you will save yourself a lot of time later on if you take a few seconds to write down notes in the margins. After you’ve finished reading a source, reflect on whether the source defends, challenges, or qualifies your argument.

You will have around 13 minutes to read through all the sources, but it’s very possible you will finish earlier if you are a fast reader. Take the leftover time to start developing your thesis and organizing your thoughts into an outline so you have more time to write. 

Step 3: Write a Strong Thesis Statement 

In order to write a good thesis statement, all you have to do is decide your stance on the claim provided in the prompt and give an overview of your evidence. You essentially have three choices on how to frame your thesis statement: You can defend, challenge or qualify a claim that’s been provided by the prompt. 

  • If you are defending the claim, your job will be to prove that the claim is correct .
  • If you are challenging the claim, your job will be to prove that the claim is incorrect .
  • If you choose to qualify the claim, your job will be to agree to a part of the claim and disagree with another part of the claim. 

A strong thesis statement will clearly state your stance without summarizing the issue or regurgitating the claim. The CollegeBoard is looking for a thesis statement that “states a defensible position and establishes a line of reasoning on the issue provided in the prompt.”

Step 4: Create a Minimal Essay Outline

Developing an outline might seem like a waste of time when you are up against the clock, but believe us, taking 5-10 minutes to outline your essay will be much more useful in the long run than jumping right into the essay.

Your outline should include your thesis statement and three main pieces of evidence that will constitute each body paragraph. Under each piece of evidence should be 2-3 details from the sources that you will use to back up your claim and some commentary on how that evidence proves your thesis.

Step 5: Write your Essay

Use the remaining 30-35 minutes to write your essay. This should be relatively easy if you took the time to mark up the sources and have a detailed outline.  Remember to add special consideration and emphasis to the commentary sections of the supporting arguments outlined in your thesis. These sentences are critical to the overall flow of the essay and where you will be explaining how the evidence supports or undermines the claim in the prompt.

Also, when referencing your sources, write the in-text citations as follows: “Source 1,” “Source 2,” “Source 3,” etc. Make sure to pay attention to which source is which in order to not incorrectly cite your sources. In-text citations will impact your score on the essay and are an integral part of the process.

After you finish writing, read through your essay for any grammatical errors or mistakes before you move onto the next essay.

Here are six must-have tips and tricks to get a good score on the synthesis essay:

  • Cite at least four sources , even though the minimum requirement is three. Remember not to plagiarize and cite everything you use in your arguments.
  • Make sure to develop a solid and clear thesis . Develop a stable stance for the claim and stick with it throughout the entire paper.
  • Don’t summarize the sources. The summary of the sources does not count as an argument. 
  • You don’t necessarily have to agree with the sources in order to cite them. Using a source to support a counterargument is still a good use of a source.
  • Cite the sources that you understand entirely . If you don’t, it could come back to bite you in the end. 
  • Use small quotes , do not quote entire paragraphs. Make sure the quote does not disrupt the flow or grammar of the sentence you write. 

ap lang sample argument essays

Discover your chances at hundreds of schools

Our free chancing engine takes into account your history, background, test scores, and extracurricular activities to show you your real chances of admission—and how to improve them.

Here is an example prompt and essay from 2019 that received 5 of the 6 total points available:

In response to our society’s increasing demand for energy, large-scale wind power has drawn attention from governments and consumers as a potential alternative to traditional materials that fuel our power grids, such as coal, oil, natural gas, water, or even newer sources such as nuclear or solar power. Yet the establishment of large-scale, commercial-grade wind farms is often the subject of controversy for a variety of reasons.

Carefully read the six sources, found on the AP English Language and Composition 2019 Exam (Question 1), including the introductory information for each source. Write an essay that synthesizes material from at least three of the sources and develops your position on the most important factors that an individual or agency should consider when deciding whether to establish a wind farm.

Source A (photo)

Source B (Layton)

Source C (Seltenrich)

Source D (Brown)

Source E (Rule)

Source F (Molla)

In your response you should do the following:

  • Respond to the prompt with a thesis presents a defensible position.
  • Select and use evidence from at least 3 of the provided sources to support your line of reasoning. Indicate clearly the sources used through direct quotation, paraphrase, or summary. Sources may be cited as Source A, Source B, etc., or by using the description in parentheses.
  • Explain how the evidence supports your line of reasoning.
  • Use appropriate grammar and punctuation in communicating your argument.

[1] The situation has been known for years, and still very little is being done: alternative power is the only way to reliably power the changing world. The draw of power coming from industry and private life is overwhelming current sources of non-renewable power, and with dwindling supplies of fossil fuels, it is merely a matter of time before coal and gas fuel plants are no longer in operation. So one viable alternative is wind power. But as with all things, there are pros and cons. The main factors for power companies to consider when building wind farms are environmental boon, aesthetic, and economic factors.

[2] The environmental benefits of using wind power are well-known and proven. Wind power is, as qualified by Source B, undeniably clean and renewable. From their production requiring very little in the way of dangerous materials to their lack of fuel, besides that which occurs naturally, wind power is by far one of the least environmentally impactful sources of power available. In addition, wind power by way of gearbox and advanced blade materials, has the highest percentage of energy retention. According to Source F, wind power retains 1,164% of the energy put into the system – meaning that it increases the energy converted from fuel (wind) to electricity 10 times! No other method of electricity production is even half that efficient. The efficiency and clean nature of wind power are important to consider, especially because they contribute back to power companies economically.

[3] Economically, wind power is both a boon and a bone to electric companies and other users. For consumers, wind power is very cheap, leading to lower bills than from any other source. Consumers also get an indirect reimbursement by way of taxes (Source D). In one Texan town, McCamey, tax revenue increased 30% from a wind farm being erected in the town. This helps to finance improvements to the town. But, there is no doubt that wind power is also hurting the power companies. Although, as renewable power goes, wind is incredibly cheap, it is still significantly more expensive than fossil fuels. So, while it is helping to cut down on emissions, it costs electric companies more than traditional fossil fuel plants. While the general economic trend is positive, there are some setbacks which must be overcome before wind power can take over as truly more effective than fossil fuels.

[4] Aesthetics may be the greatest setback for power companies. Although there may be significant economic and environmental benefit to wind power, people will always fight to preserve pure, unspoiled land. Unfortunately, not much can be done to improve the visual aesthetics of the turbines. White paint is the most common choice because it “[is] associated with cleanliness.” (Source E). But, this can make it stand out like a sore thumb, and make the gargantuan machines seem more out of place. The site can also not be altered because it affects generating capacity. Sound is almost worse of a concern because it interrupts personal productivity by interrupting people’s sleep patterns. One thing for power companies to consider is working with turbine manufacturing to make the machines less aesthetically impactful, so as to garner greater public support.

[5] As with most things, wind power has no easy answer. It is the responsibility of the companies building them to weigh the benefits and the consequences. But, by balancing economics, efficiency, and aesthetics, power companies can create a solution which balances human impact with environmental preservation.

More examples can be found here at College Board.

While AP Scores help to boost your weighted GPA, or give you the option to get college credit, AP Scores don’t have a strong effect on your admissions chances . However, colleges can still see your self-reported scores, so you might not want to automatically send scores to colleges if they are lower than a 3. That being said, admissions officers care far more about your grade in an AP class than your score on the exam.

Related CollegeVine Blog Posts

ap lang sample argument essays

Calculate for all schools

Your chance of acceptance, your chancing factors, extracurriculars, ap language sample essays.

Hey everyone, I'm currently preparing for the AP Language exam and was wondering if you could share some good sample essays or resources where I can find high-quality examples that really showcase effective argumentation and writing. Thanks!

Hi! It's great that you're looking for sample essays to prepare for the AP Language exam. Reviewing high-quality examples can certainly help improve your understanding of effective argumentation and writing. Here are a few resources to consider:

1. College Board: The official website for AP Exams offers sample essays and scoring guidelines for previous years' questions. You can find specific examples of argumentative, synthesis, and rhetorical analysis essays, along with detailed explanations of the scoring rubric. Visit their website and search for "AP Language free response" to access these materials.

2. AP Central: Another College Board resource, AP Central, provides numerous sample essays along with scoring commentary so you can see how each essay met the criteria set by the examiners. Explore the AP Lang sections of AP Central to find relevant articles: https://apcentral.collegeboard.org/

3. CollegeVine: This platform offers guidance and resources on a variety of topics related to college admissions, including AP exams. They have a blog post series that covers strategies for success on AP Lang, and you might find some essay samples there as well. You can click here: https://blog.collegevine.com/how-to-write-ap-lang-synthesis-essay/

4. JSTOR: Your school may have a subscription for JSTOR, which is a research database containing academic journals, books, and primary sources. You can access high-quality argumentative essays across different disciplines, helping you observe effective techniques used by various writers.

5. Google Scholar: Similar to JSTOR, Google Scholar can be used to search for academic resources and peer-reviewed papers. You might find well-constructed argumentative essays that provide valuable insights into writing styles and argumentation methods.

Don't forget to analyze not just the content but also the structure and strategies employed in these essays. Focus on identifying what makes them effective, and apply those techniques to your own writing. Good luck with your AP Language preparations!

About CollegeVine’s Expert FAQ

CollegeVine’s Q&A seeks to offer informed perspectives on commonly asked admissions questions. Every answer is refined and validated by our team of admissions experts to ensure it resonates with trusted knowledge in the field.

200+ Transition Words for Argumentative Essay

Transition Words for Argumentative Essay

Transition words are linking words or phrases that connect ideas, sentences, or paragraphs, ensuring a smooth flow of information. They help improve coherence and guide readers through the text.

In this article, let’s talk about what good transition words for argumentative essays are, why they matter so much in argumentative essays, and how to use them effectively. You'll get a clear idea of how these words can make your writing more convincing and easier to read. Plus, we’ll provide you with plenty of examples so you can see how to use them in your own essays.

And if you finish reading and still feel like you could use a hand with your essay, DoMyEssay is here to help. We can assist with any part of your writing process, whether it’s refining your arguments or polishing the final draft.

How to Use Transition Words for Argumentative Essays?

Let’s say you’re in the middle of crafting your argumentative essay. You’ve got great points, and everything makes sense in your head, but when you read it back, it feels a bit disjointed. That’s where transition words come in. When used right, the appropriate transition words can act as a hook for argumentative essay, keeping your readers engaged.

Here’s how to use them effectively:

  • Give Each Transition Word a Job Every transition word has a specific role in your essay. Some introduce a new idea, like "firstly" or "to begin with." If you’re adding more info, you might use "also" or "in addition." Then there are those that contrast ideas, like "however" or "on the other hand." Knowing what each word is meant to do helps you pick the right one for the right moment.
  • Use Them Where They Count You want to place transition words where they’ll have the most impact (usually at the start of a new paragraph or when you’re shifting to a new point). For example, if you’ve just argued that renewable energy is good for the environment, you could say, "For instance," before giving a specific example. 
  • Keep Your Essay Smooth The whole point of using transition words is to keep your essay coherent and flowing naturally. For example, after discussing one advantage of a policy, you might use "similarly" to transition smoothly to the next point, making sure your essay feels connected and logical.
  • Mix It Up Using the same transition word over and over can get boring fast. So, try to vary them: use "moreover" instead of always saying "also," or "on the flip side" instead of "however." This keeps your writing interesting and shows that you have a good range of vocabulary. But don’t overdo it: too many transitions can make your essay feel forced.
  • Play with Placement Well, transition words don’t always have to go at the beginning of a sentence. Sometimes, they work in the middle or even at the end. For example, you could start a sentence with "Therefore" to show a conclusion. Or, you could use it in the middle: "The policy is effective, therefore reducing costs." This gives your writing a nice rhythm!

Transform Your Essay Today!

Struggling with transitions or arguments in your essay? Let us help you make your essay shine.

ap lang sample argument essays

Types of Argumentative Essay Transition Words

Transition words connect your ideas in an argumentative essay, helping your reader follow along without getting lost. They’re small tools that can make your already persuasive writing clearer and more persuasive. Let’s break down some different types of transition words you can use.

Starting a New Point

When you’re starting a new idea or argument, you want your reader to know you’re shifting to something new. Transition words for essays like "initially" are great for signaling this. These phrases help set the stage for the new point you’re about to make, letting your reader know what’s coming next:

  • "To start with, renewable energy is a cleaner alternative to fossil fuels."
  • "Firstly, social media has changed how we interact with each other."

Adding More Information

Sometimes you need to pile on additional info to back up your argument. Transition words like "besides" help you do just that and make it easy to add layers to your argument, giving it more depth:

  • "Furthermore, renewable energy sources can create new jobs in the economy."
  • "Also, social media platforms are becoming key tools in education."

Giving Examples

When you want to make your point more concrete, you’ll need to throw in some examples. Words like "for example" help you introduce these examples smoothly. These transitions show your readers that you’re about to give them something real to back up your claim:

  • "For instance, countries like Germany have heavily invested in solar power."
  • "To illustrate, many businesses use social media to connect directly with customers."

Showing Contrast

Not all points will be in agreement with each other, and that’s okay. When you need to highlight differences or opposing ideas, words like "in contrast" can be your go-tos. These phrases help you present complex ideas with a balanced view by acknowledging other perspectives:

  • "However, some argue that renewable energy is still too expensive to implement widely."
  • "On the other hand, social media can sometimes lead to misinformation spreading quickly."

Explaining Cause and Effect

When you need to show that one thing leads to another, transition words like "therefore" help make that connection clear. These transitional words help you link your ideas, showing how one thing influences another:

  • "Because of the increase in renewable energy, carbon emissions have significantly decreased."
  • "As a result, social media has become a powerful tool for activism."

Backing Up with Evidence

When you’re supporting your argument with facts or research, transition words like "according to" or "as shown by" help you introduce this evidence clearly and lend credibility to your argument:

  • "According to recent studies, renewable energy could power 80% of the U.S. by 2050."
  • "In fact, social media use has been linked to increased engagement in political activities."

Addressing the Other Side

A good argumentative essay doesn’t ignore opposing views. Transition words like "despite" help you address these counterarguments and show that you’ve considered the key point from all angles, which strengthens your argument:

  • "Although renewable energy has a higher initial cost, the long-term benefits outweigh it."
  • "Even though social media has its downsides, its benefits cannot be ignored."

Wrapping It All Up

Finally, when you’re ready to conclude your essay, use words like "to sum up" to bring everything together and signal to your reader that you’re wrapping things up and summarizing your main points:

  • "In conclusion, shifting to renewable energy is not just an option, but a necessity for a sustainable future."
  • "All in all, social media's impact on society is profound, offering both opportunities and challenges."

List of Transition Words for Argumentative Essays

Introduction.

  • To begin with
  • In the first place
  • First and foremost
  • At the outset
  • To start with
  • As an introduction
  • Before anything else
  • Let’s start by
  • The first point is
  • It all starts with
  • To introduce the topic
  • To open the discussion
  • In the beginning
  • To set the stage
  • Starting with
  • In the first instance
  • To lay the foundation
  • As we start
  • To kick things off
  • The first thing to address is
  • As a starting point
  • Let’s first consider
  • Additionally
  • Furthermore
  • In addition
  • Not to mention
  • What’s more
  • On top of that
  • Equally important
  • Another point is
  • Beyond that
  • To further elaborate
  • Coupled with
  • For example
  • For instance
  • To illustrate
  • As an example
  • Specifically
  • To give you an idea
  • Take the case of
  • Consider the following
  • As illustrated by
  • In particular
  • In this case
  • To demonstrate
  • As shown by
  • One example is
  • To put it in perspective
  • An illustration of this is
  • A good example is

Contradiction

  • On the other hand
  • Nevertheless
  • Nonetheless
  • In contrast
  • On the contrary
  • Despite this
  • In spite of
  • Even though

Cause and Effect

  • As a result
  • Consequently
  • Because of this
  • For this reason
  • Accordingly
  • As a consequence
  • This leads to
  • Resulting in
  • In response to
  • With this in mind
  • As evidence
  • According to
  • As demonstrated by
  • In support of this
  • Research shows
  • To back this up
  • As confirmed by
  • As indicated by
  • As corroborated by
  • Studies reveal
  • As evidenced by
  • Data suggests
  • As supported by
  • Undoubtedly
  • It is evident that
  • There is no denying that
  • It is clear that
  • It is certain that
  • To emphasize
  • As a matter of fact
  • Unquestionably
  • This suggests that
  • It stands to reason that
  • In light of this
  • This demonstrates
  • This proves

Counterargument

  • It is true that
  • One might argue
  • Some may say
  • While it is true
  • Although it is often believed
  • On the one hand
  • This may be true, but
  • There is some truth to
  • In conclusion
  • All things considered
  • To conclude
  • As a final point
  • Taking everything into account
  • In the final analysis
  • Upon reflection
  • In retrospect
  • To summarize
  • Considering these points
  • As has been demonstrated

Clarification

  • In other words
  • To put it simply
  • That is to say
  • In simpler terms
  • To put it another way
  • What this means is
  • For clarity
  • To rephrase
  • Put differently
  • To be clear
  • More specifically
  • In plain language
  • To put it clearly
  • Stated differently
  • To break it down
  • To put things into perspective
  • Without a doubt
  • Significantly
  • Particularly
  • Most importantly
  • Emphatically
  • It should be emphasized
  • It is worth noting
  • Importantly
  • Subsequently
  • Following this
  • Simultaneously
  • In the meantime
  • In the same way
  • In a similar manner
  • In like manner
  • By the same token
  • In a similar fashion
  • Correspondingly
  • In parallel
  • Comparable to
  • In the same vein
  • Analogous to
  • In the same spirit
  • Along similar lines
  • In much the same way

Simple Help for Better Essays

Want to improve your essay’s flow and impact? We’re here to help with easy-to-follow advice and edits.

ap lang sample argument essays

Wrapping Up

We’ve discussed what the different transition words are, why they matter, and how to use them effectively, complete with examples and tips.

To help you even further, here’s a handy table summarizing the best transition sentences and words to use for each part of your body paragraphs, with some more examples:

Essay Part Transition Words Examples
To kick off, As a starting point, In the beginning, Let's start with, The first aspect
Moving forward, In addition, On the next note, Expanding on this, Building upon that
Beyond that, Additionally, On another level, What's more, Not to mention
Wrapping up, Finally, As a final thought, Bringing this to a close, In conclusion
On the flip side, Conversely, That said, From a different perspective, On the contrary

On the flip side, Conversely, That said, From a different perspective, On the contrary

But if you’re still scratching your head after reading this, wondering how to add these transitions into your essay, DoMyEssay can help with any part of your writing process. Whether you need assistance with transitions, crafting strong and convincing arguments, or polishing your final draft, we’ve got you covered.

ap lang sample argument essays

AP English Language and Composition

Review the free-response questions from the 2024 ap exam, new for 2024-25: mcqs will have four answer choices.

Starting in the 2024-25 school year, AP English Language and Composition multiple-choice questions (MCQs) will have four answer choices instead of five. This change will take effect with the 2025 exam. All resources have been updated to reflect this change.

Exam Overview

Exam questions assess the course concepts and skills outlined in the course framework. For more information, download the  AP English Language and Composition Course and Exam Description (.pdf)  (CED).

Encourage your students to visit the  AP English Language and Composition student page  for exam information.

Wed, May 14, 2025

AP English Language and Composition Exam

Exam format.

The AP English Language and Composition Exam has question types and point values that stay consistent from year to year, so you and your students know what to expect on exam day.

Section I: Multiple Choice

45 Questions | 1 hour | 45% of Exam Score

  • 23–25 Reading questions that ask students to read and analyze nonfiction texts.
  • 20–22 Writing questions that ask students to “read like a writer” and consider revisions to stimulus texts.

Section II: Free Response

3 Questions | 2 hours 15 minutes (includes a 15-minute reading period | 55% of Exam Score

  • Synthesis Question:  After reading 6 texts about a topic (including visual and quantitative sources), students will compose an argument that combines and cites at least 3 of the sources to support their thesis.
  • Rhetorical Analysis:  Students will read a nonfiction text and analyze how the writer’s language choices contribute to the intended meaning and purpose of the text.
  • Argument:  Students will create an evidence-based argument that responds to a given topic.

Exam Questions and Scoring Information

Ap english language and composition exam questions and scoring information.

View free-response questions and scoring information from past exams.

Score Reporting

Ap score reports for educators.

Access your score reports.

COMMENTS

  1. AP English Language and Composition Exam Questions

    Download free-response questions from this year's exam and past exams along with scoring guidelines, sample responses from exam takers, and scoring distributions. If you are using assistive technology and need help accessing these PDFs in another format, contact Services for Students with Disabilities at 212-713-8333 or by email at ssd@info ...

  2. How to Write the AP Lang Argument Essay (With Example)

    Typically, the AP Lang Argument Essay prompt asks you to reflect on a broad cultural, moral, or social issue that is open to debate. For evidence, you won't be asked to memorize and cite statistics or facts. Rather, you'll want to bring in real-world examples of: Historical events. Current-day events from the news.

  3. How to Write the AP Lang Argument Essay + Examples

    Learn how to structure, organize, and write a persuasive argumentative essay for the AP English Language Exam. See tips, tricks, and sample essays for different prompts and topics.

  4. PDF AP English Language and Composition

    Learn how to write an effective argument essay for the AP English Language and Composition exam. See the scoring criteria, decision rules, and examples for each reporting category: thesis, evidence and commentary, and sophistication.

  5. PDF ap06 english lang student samples

    The question directed students to read carefully an excerpt of William Hazlitt's 1827 essay, "On the Want of Money," and to analyze the rhetorical strategies the author uses to develop his position about money. Sample: 2A Score: 8. This essay's control and focus are evident from the first sentence.

  6. PDF AP English Language and Composition

    Provide evidence to support your line of reasoning. Explain how the evidence supports your line of reasoning. Use appropriate grammar and punctuation in communicating your argument. 2023 College Board. Reporting Category. Scoring Criteria. Row A Thesis (0-1 points) 0 points. For any of the following:

  7. PDF AP English Language and Composition Question 3: Argument (2019) Sample

    See how students argued that capitalism and the Electoral College are overrated in their AP Lang FRQ responses. The essays are scored with the 2020 rubrics and include commentaries.

  8. PDF AP® ENGLISH LANGUAGE AND COMPOSITION

    1 - Essays earning a score of 1 meet the criteria for the score of 2 but are undeveloped, especially simplistic in their explanation and argument, weak in their control of language, or especially lacking in coherence and development. 0 - Indicates an off-topic response, one that merely repeats the prompt, an entirely crossed-out response, a ...

  9. How to Craft an Argument for AP® English Language

    How to Craft an Argument for AP® English Language. The AP® English Language persuasive (or argumentative) essay is one of the three long-form free-response questions that will make up 55% of your score on the AP® English Language and Composition Exam. While the multiple-choice section and the rhetorical analysis essay will test you on how ...

  10. Crafting an Impressive Argumentative Essay for AP Lang

    4. Organize Your Essay Effectively: - Tip: Structure your essay with a clear introduction, body paragraphs, and a conclusion. Ensure a logical flow of ideas, with each paragraph contributing to the overall argument. 5. Provide Context and Background: - Tip: Begin with a brief introduction that provides context for your argument.

  11. How to Get a 6 on Argument FRQ in AP® English Language

    Pick an opinion and stick to it. Choose one side of the argument and one clear claim to support all the way through. Craft a thesis statement. Your thesis should be clear, concise, and introduce the content of your essay. Craft a chronological argument. Make an argument that builds on its prior points.

  12. What's the best approach for the AP Lang argument essay?

    The AP Lang argument essay is all about making a strong, well-structured argument in a clear and concise manner. Here's a step-by-step approach you can follow and some tips to help you prepare: 1. Understand the prompt: Carefully read the given prompt and make sure you understand what it's asking you to do. Keep an eye out for key terms that ...

  13. AP Lang sample argument essays

    Hey there! You're on the right track by looking at real examples to improve your AP Lang argument essays. The College Board website is a great place to start, as they provide sample essays for past prompts and even include scoring guidelines.

  14. PDF AP English Language and Composition

    AP® English Language and Composition 2022 Scoring Guidelines. Argument Essay 6 points . Colin Powell, a four-star general and former United States secretary of state, wrote in his 1995 autobiography: "[W]e do not have the luxury of collecting information indefinitely. At some point, before we can have every possible fact in hand, we have to ...

  15. AP English Language Essay Examples

    Hello! It's a great idea to learn from high-scoring essay examples as it helps you understand what's expected and refine your writing skills. One of the best resources for sample essays is the College Board website, which provides essay samples from actual AP English Language and Composition exams. These samples include scoring guidelines and explanations, so you can see how each essay meets ...

  16. PDF AP English Language and Composition

    6 - Adequate. Essays earning a score of 6 adequately develop a position on the value of exploring the unknown. The evidence and explanations appropriately and sufficiently support the student's position, and the argument is coherent and adequately developed. The writing may contain lapses in diction or syntax, but generally the prose is clear.

  17. AP Lang Argument Essay: Evidence

    Unit 1 - Claims, Reasoning, & Evidence. Unit 2 - Organizing Information for a Specific Audience. Unit 3 - Perspectives & How Arguments Relate. Unit 4 - How writers develop arguments, intros, & conclusions. Unit 5 - How a writer brings all parts of an argument together. Unit 6 - Position, Perspective, & Bias. Unit 7 - Successful ...

  18. Expert Guide to the AP Language and Composition Exam

    So each of the three free-response essays is worth about 18% of your score. As on other APs, your raw score will be converted to a scaled score of 1-5. This exam has a relatively low 5 rate. Only 10% of test takers received a 5 in 2022, although 56% of students received a score of 3 or higher.

  19. PDF AP English Language and Composition

    Write an essay that synthesizes material from at least three of the sources and develops your position on the value, if any, of initiatives to improve STEM education and increase the number of students ... • Use appropriate grammar and punctuation in communicating your argument. AP® English Language and Composition 2022 Scoring Guidelines ...

  20. How to Write the AP Lang Synthesis Essay + Example

    Step 5: Write your Essay. Use the remaining 30-35 minutes to write your essay. This should be relatively easy if you took the time to mark up the sources and have a detailed outline. Remember to add special consideration and emphasis to the commentary sections of the supporting arguments outlined in your thesis.

  21. PDF ap06 english lang student samples

    This web page provides the scoring criteria and commentary for Question 3 of the 2006 AP English Language and Composition exam. It includes sample essays and explanations for each score level, from 1 to 9.

  22. AP Language sample essays?

    1. College Board: The official website for AP Exams offers sample essays and scoring guidelines for previous years' questions. You can find specific examples of argumentative, synthesis, and rhetorical analysis essays, along with detailed explanations of the scoring rubric. Visit their website and search for "AP Language free response" to ...

  23. 200+ Essential Transition Words for Argumentative Essay

    Types of Argumentative Essay Transition Words Transition words connect your ideas in an argumentative essay, helping your reader follow along without getting lost. They're small tools that can make your already persuasive writing clearer and more persuasive. Let's break down some different types of transition words you can use.

  24. AP English Language and Composition Exam

    Section I: Multiple Choice. 45 Questions | 1 hour | 45% of Exam Score. Includes 5 sets of questions: 23-25 Reading questions that ask students to read and analyze nonfiction texts. 20-22 Writing questions that ask students to "read like a writer" and consider revisions to stimulus texts.